B & D????
I don't see how B and D differ. They both ultimately say that the field inspectors may have some ...
JJennings on April 20, 2020
  • December 2009 LSAT
  • SEC3
  • Q20
8
Replies
Why not C?
Can someone explain to me why c is incorrect?
nivensdc on February 18, 2020
  • December 2009 LSAT
  • SEC3
  • Q22
1
Reply
Why not B?
Is B wrong because it weakens Gilbert's argument, but does not strengthen Sabina's? Thanks!
zacdon81 on February 18, 2020
  • December 2009 LSAT
  • SEC3
  • Q5
1
Reply
D please
Please explain why D is incorrect. It seems to rule out an alternative cause I.e that vacation wa...
amf on February 10, 2020
  • December 2009 LSAT
  • SEC3
  • Q22
1
Reply
Why Not C?
Why isn't C the correct answer?
nivensdc on November 30, 2019
  • December 2009 LSAT
  • SEC3
  • Q4
1
Reply
Can you explain this?
Can you explain this one?
nivensdc on November 30, 2019
  • December 2009 LSAT
  • SEC3
  • Q14
1
Reply
Question about C
How are we supposed to cross out C for being wrong? My thought process: -Marthus's argument: ...
CHLee on November 14, 2019
  • December 2009 LSAT
  • SEC3
  • Q18
2
Replies
B
So even though there is significantly less methane in the earth's atmosphere today than there was...
tomgbean on October 22, 2019
  • December 2009 LSAT
  • SEC3
  • Q25
1
Reply
E
You probably hear this question a lot...but, the answer choice that strengthens the argument does...
tomgbean on October 22, 2019
  • December 2009 LSAT
  • SEC3
  • Q5
1
Reply
please need help!
What is the difference between B and C?
ali on October 12, 2019
  • December 2009 LSAT
  • SEC3
  • Q19
1
Reply
Why B?
I do not understand why mentioning methane in the argument leads to it be a considerable part of ...
NicoCapri on September 22, 2019
  • December 2009 LSAT
  • SEC3
  • Q25
2
Replies
Help
Please explain this question
mechirenee on September 4, 2019
  • December 2009 LSAT
  • SEC3
  • Q16
4
Replies
B or C?
Why is C a better choice than B?
ariella on July 23, 2019
  • December 2009 LSAT
  • SEC3
  • Q13
3
Replies
C
can you explain why C is incorrect please?
ariella on July 23, 2019
  • December 2009 LSAT
  • SEC3
  • Q21
5
Replies
Argument Completion or Argument Structure
Is this filed under the right category?
rolltribe on July 18, 2019
  • December 2009 LSAT
  • SEC3
  • Q12
6
Replies
Why is E Correct?
This does not make much sense to me, I am missing the assumption
zgnewquist on July 12, 2019
  • December 2009 LSAT
  • SEC3
  • Q5
1
Reply
Why is E wrong?
Could someone please explain this argument, and why E is wrong?
SarahA on May 6, 2019
  • December 2009 LSAT
  • SEC3
  • Q8
3
Replies
Question for why E is wrong and A is correct
Hi, It is concluded that these foods CAUSE parkinsons, but nothing is mentioned on its effect ...
Maroun on March 17, 2019
  • December 2009 LSAT
  • SEC3
  • Q1
1
Reply
question
Why is the answer not A?
claire_crites on February 12, 2019
  • December 2009 LSAT
  • SEC3
  • Q14
1
Reply
Why is A right and D is wrong?
I don't understand
Jessica-Killeen on January 22, 2019
  • December 2009 LSAT
  • SEC3
  • Q1
1
Reply